LSAT and Law School Admissions Forum

Get expert LSAT preparation and law school admissions advice from PowerScore Test Preparation.

 Administrator
PowerScore Staff
  • PowerScore Staff
  • Posts: 8916
  • Joined: Feb 02, 2011
|
#41173
Complete Question Explanation
(The complete setup for this game can be found here: lsat/viewtopic.php?t=15645)

The correct answer choice is (E)

When L is harvested, then according to the second and third rules J, K, and M must all be harvested after L, with no restriction on the placement of J in the last three spaces:
  • pt48_d05_g2_q12.png
Accordingly, answer choice (E) is correct.
You do not have the required permissions to view the files attached to this post.
 sarahmil
  • Posts: 13
  • Joined: Jan 29, 2019
|
#73566
I’m confused by this question. I had trouble eliminating the answers. I thought a lot of them could be true. I chose B. But I also thought C could work.

Cuz I thought the following could occur:

(P , G , H-J) L (K , T, M)

So b and c work

Or (P , G , H/T) L (K , M, H/J)
User avatar
 Dave Killoran
PowerScore Staff
  • PowerScore Staff
  • Posts: 5853
  • Joined: Mar 25, 2011
|
#73568
sarahmil wrote:I’m confused by this question. I had trouble eliminating the answers. I thought a lot of them could be true. I chose B. But I also thought C could work.

Cuz I thought the following could occur:

(P , G , H-J) L (K , T, M)

So b and c work
The problem above is that from the second rule L is ahead of J, but your diagram has that reversed. So J is after L, and T will have to be before L.


sarahmil wrote:Or (P , G , H/T) L (K , M, H/J)
This diagram has only three variables ahead of L, but since L is fifth there must be four, so it won't work either :(

Please let me know if that helps. Thanks!
 dimi.wassef@yahoo.com
  • Posts: 34
  • Joined: Aug 26, 2021
|
#92911
I do not understand this question at all. I came to the conclusion that the sequence P/G H J L* K/T M meets the conditions of each rule, yet both B and C are obviously wrong answers? Can someone help me understand this?
 Robert Carroll
PowerScore Staff
  • PowerScore Staff
  • Posts: 1787
  • Joined: Dec 06, 2013
|
#92914
dimi.wassef,

J must be after L, so that solution does not work.

Robert Carroll

Get the most out of your LSAT Prep Plus subscription.

Analyze and track your performance with our Testing and Analytics Package.